0 Daumen
312 Aufrufe

Aufgabe:


\( \lim\limits_{x\to\infty} \) (1+\( \frac{1}{3x} \))^x


Problem/Ansatz:

ich wollte mal Fragen, ob mein Ansatz/ Lösung, die ich habe so stimmt. Bin nach dem Lösungsverfahren von L'Hospital gegangen.

\( \lim\limits_{x\to\infty} \) ex*ln(1+\( \frac{1}{3x} \)) 

\( \lim\limits_{x\to\infty} \) \( \frac{ln(1+\frac{1}{3x})}{\frac{1}{x}} \)


Bin ich auf dem richtigen Weg oder stimmt mein Ergebnis sogar?


:)

Avatar von

2 Antworten

0 Daumen

Ich würde hier allerdings eher über die Def. der Exp.fkt gehen

(1 + t/x)^x geht für x gegen unendlich gegen e^t.

Also ( 1 + (1/3) / x ) ^x geht gegen e^(1/3) .

Mit Hospital geht es auch, bei deinem Term gibt die

Abl. von Zähler und Nenner dann ja

( -1/(x*(3x+1) ) /  ( -1 / x^2 ) =  x^2 / ( (x*(3x+1) ) =  x / (3x+1)  =  1 ( 3 + 1/x )

und das geht auch gegen 1/3.

Avatar von 288 k 🚀

Vielen Dank!

Das leuchtet mir jetzt ein :). Der Weg ist ist einfacher.

0 Daumen

Das Ergebnis ist die dritte Wurzel aus e.

Das siehst du vielleicht, wenn du den Exponenten x als \( 3x\cdot \frac{1}{3} \) schreibst.

Avatar von 53 k 🚀

Vielen Dank!

Habe es nun verstanden :)

Ein anderes Problem?

Stell deine Frage

Willkommen bei der Mathelounge! Stell deine Frage einfach und kostenlos

x
Made by a lovely community